Nalmed Province

This topic has expert replies
Master | Next Rank: 500 Posts
Posts: 110
Joined: Mon Feb 09, 2015 1:52 am
Followed by:4 members

Nalmed Province

by Abhijit K » Mon Mar 23, 2015 10:52 pm
Which of the following, if true, most logically completes the passage?

A recent poll found that over 80 percent of the residents of Nalmed Province favored a massive expansion of the commuter rail system as a means of significantly easing congestion on the province's highways and were willing to help pay for the expansion through an increase in their taxes. Nevertheless, the poll results indicate that expansion of the rail system, if successfully completed, would be unlikely to achieve its goal of easing congestion, because _______.

A. most people in favor of expanding the rail system reported less congestion during their highway commute as the primary benefit they would experience
B. of the less than 20 percent of residents not counted as favoring the expansion, about half claimed to have no opinion one way or the other
C. the twice-daily periods of peak congestion caused by people commuting in cars have grown from about an hour each to almost two and a half hours each in the past 20 years
D. expanding the commuter rail system will require the construction of dozens of miles of new railbed
E. the proposed expansion to the commuter rail system will make it possible for some people who both live and work at suburban locations to commute by rail

User avatar
Junior | Next Rank: 30 Posts
Posts: 21
Joined: Thu Mar 05, 2015 12:01 pm
Location: Boston, MA
Thanked: 10 times
Followed by:3 members
GMAT Score:770

by Spencer@Prep4GMAT » Tue Mar 24, 2015 10:39 am
Hi Abhijit,

The logic of the proposed rail expansion to ease congestion is as follows: (1) Expand rail system --> (2) People use the rail instead of driving on the highways --> (3) Less highway congestion.

On the other hand, the argument (final sentence) states that something in the poll results indicates that accomplishing (1) is unlikely to accomplish (3).

How might accomplishing (1) not lead to (3)? This would be the case if (2) does not happen. In other words, the rail system is built, BUT most people continue to drive instead of using the rail and the highways stay congested as a result.

In answer (A), we more or less find this idea: "Most" of the residents favoring the rail proposal do so because they believe OTHER people will use the rail rather than drive, leaving the highways less congested. They, however, plan to continue driving to work! If that's the case for a majority of residents, then (2) in the logical chain of events will indeed not happen, breaking the chain from (1) to (3).

Does that make sense? Answers (B) through (E) are largely irrelevant.

Hope that helps!
Ready4